Given u = (3, 1, -2), v = (2,0,−1), w = (–1,1,2)

Find the cosine of the angle between the vectors v=<2,6,9> and w=<3,-2,6>

Answers

Answer 1

The cosine of the angle between the vectors v=<2,6,9> and w=<3,-2,6> is approximately 0.878.

To find the cosine of the angle between two vectors, we can use the dot product formula. Let v=<2,6,9> and w=<3,-2,6> be the given vectors.

Step 1: Calculate the dot product of v and w: v · w = (2)(3) + (6)(-2) + (9)(6) = 6 - 12 + 54 = 48.

Step 2: Calculate the magnitudes of v and w: |v| = sqrt(2^2 + 6^2 + 9^2) ≈ 10.677 and |w| = sqrt(3^2 + (-2)^2 + 6^2) ≈ 7.616.

Step 3: Apply the cosine formula: cosθ = (v · w) / (|v| |w|) = 48 / (10.677 * 7.616) ≈ 0.878.

Therefore, the cosine of the angle between the vectors v=<2,6,9> and w=<3,-2,6> is approximately 0.878.

Learn more about vectors : brainly.com/question/24256726

#SPJ11


Related Questions

(1) Show all the steps of your solution and simplify your answer as much as possible. (2) The answer must be clear, intelligible, and you must show your work. Provide explanation for all your steps. Your grade will be determined by adherence to these criteria. 2 Evaluate the following integral: ₂2-1²(x²+1) dx.

Answers

The evaluated integral is \[\boxed{\frac{1}{2}\ln10-\frac{1}{2}\ln2}\] which is a proper solution to this question.

We have to evaluate the following integral: \[\int_{2}^{1}(x^{2}+1)(2-x^{2})dx\] This integral can be evaluated by the method of substitution. Substituting the term, \[(2-x^{2})\]as t, we get\[t=2-x^{2}\]Differentiating both sides, we get\[dt/dx=-2x\]Solving for dx, we get \[dx=-dt/2x\] The limits of integration are 2 and 1, which on substitution give\[t_{1}=2-1^{2}=1\]and\[t_{2}=2-2^{2}=-2\] The integral can now be expressed as\[\int_{1}^{-2}(x^{2}+1)\frac{-dt}{2x}\] Simplifying this, we get\[-\frac{1}{2}\int_{1}^{-2}\frac{(x^{2}+1)}{x}dt\].

Solving the integral by partial fractions, we get\[-\frac{1}{2}\int_{1}^{-2}\left ( \frac{1}{x}-\frac{x}{x^{2}+1} \right )dt\] We can now evaluate the integral as\[-\frac{1}{2} \left [ \ln |x| - \frac{1}{2}\ln (x^{2}+1) \right ]_{1}^{-2}\]On substituting the limits of integration, we get\[\frac{1}{2}(\ln 2+\ln 5)\]Simplifying, we get the answer as\[\boxed{\frac{1}{2}\ln10-\frac{1}{2}\ln2}\] Therefore, the evaluated integral is \[\boxed{\frac{1}{2}\ln10-\frac{1}{2}\ln2}\] which is a proper solution to this question.

To know more about integral visit:-

https://brainly.com/question/31433890

#SPJ11


susan moved to the inner city 7years ago. the population was 30,000
at the time. the population is now 45,000. calculate the
appropriate mean rate of growth over this period of 7 years.

Answers

To calculate the mean rate of growth over a period of 7 years, we need to find the average annual growth rate. The formula to calculate the average annual growth rate is:

Mean Growth Rate = (Final Population / Initial Population)^(1/Number of Years) - 1

Given:

Initial Population (P0) = 30,000

Final Population (P7) = 45,000

Number of Years (n) = 7

Plugging in these values into the formula, we can calculate the mean rate of growth:

Mean Growth Rate = (45,000 / 30,000)^(1/7) - 1

Calculating this expression:

Mean Growth Rate = (1.5)^(1/7) - 1

≈ 0.0906

Therefore, the appropriate mean rate of growth over the period of 7 years is approximately 0.0906, or 9.06%. This means that, on average, the population has been growing at a rate of 9.06% per year over the past 7 years.

Learn more about statistics here:

https://brainly.com/question/30233021

#SPJ11

Determine if the data sets A and B are independent. Do these data A 65 68 96 55 92 69 89 71 40 91 43 54 91 47 51 88 84 data B 50 96 82 81 90 84 87 97 69 54 80 85 99 55 53 60 51 Bsets have the same mean? If the data sets are dependent find the equation of the regression line connecting these two data sets.

Answers

To determine if the data sets A and B are independent, we need to analyze the relationship between the two sets.

To determine if the data sets A and B are independent, we can examine their relationship. If there is no apparent relationship or correlation between the data sets, they can be considered independent. If there is a relationship between the data sets, they are dependent.

To find the means of both data sets, we sum up the values in each set and divide by the number of observations. For data set A, the mean is (65+68+96+55+92+69+89+71+40+91+43+54+91+47+51+88+84)/17 = 71.47. For data set B, the mean is (50+96+82+81+90+84+87+97+69+54+80+85+99+55+53+60+51)/17 = 74.18.

Since the means of data sets A and B are different (71.47 ≠ 74.18), we can conclude that the data sets are not the same.

As the data sets are not independent and have a relationship, we can find the equation of the regression line connecting them.

Learn more about sets here:

https://brainly.com/question/28492445

#SPJ11

The integral J dx/3√x + √x
can be rewritten as
(a) J 6u^3/u + 1 du
(b) J 6u^2/u^2 + 1 du
(c) J 6u^4/u^2 + 1 du
(d) J 6u^5/u^3 + 1 du

Answers

To rewrite the integral ∫ dx / (3√x + √x), we can simplify the denominator by combining the two square roots:

√x = √x * √x = √(x^2) = |x|

Therefore, the integral becomes:

∫ dx / (3√x + √x) = ∫ dx / (3|x| + |x|)

Now, we can factor out |x| from the denominator:

∫ dx / (3|x| + |x|) = ∫ dx / (4|x|)

Now, we need to consider the absolute value of x. Depending on the sign of x, we have two cases:

For x ≥ 0:

In this case, |x| = x, so the integral becomes:

∫ dx / (4x) = 1/4 ∫ dx / x = 1/4 ln|x| + C

For x < 0:

In this case, |x| = -x, so the integral becomes:

∫ dx / (4(-x)) = -1/4 ∫ dx / x = -1/4 ln|x| + C

Therefore, the rewritten integral is:

∫ dx / (3√x + √x) = 1/4 ln|x| + C

So the correct choice is (a) ∫ 6u^3 / (u + 1) du.

To learn more about denominator : brainly.com/question/19372232

#SPJ11

Find parametric equations for the following curve. Include an interval for the parameter values. The complete curve x = -5y3 - 3y Choose the correct answer below. O A. x=t, y= - 513 - 3t - 7sts5 B. x=t, y= - 513 - 3t; -00

Answers

The parametric equations for the curve are:

x = -5t^3 - 3t

y = t

To find parametric equations for the curve x = -5y^3 - 3y, we can set y as the parameter and express x in terms of y.

Let y = t, where t is the parameter.

Substituting y = t into the equation x = -5y^3 - 3y:

x = -5(t^3) - 3t

The interval for the parameter values depends on the context or specific requirements of the problem. If no specific interval is given, we can assume a wide range of values for t, such as all real numbers.

So, the correct answer is:

A. x = -5t^3 - 3t, y = t

Know more about parametric equations here:

https://brainly.com/question/29275326

#SPJ11

Let n1=80, X1=20, n2=100, and X2=10. The value of P_1 ,P_2
are:
0.4 ,0.20
0.5 ,0.20
0.25, 0.10
0.5, 0.25

Answers

Let n1 = 80, X1 = 20, n2 = 100, and X2 = 10P_1 and P_2 values are 0.25 and 0.10

Given n1 = 80, X1 = 20, n2 = 100, and X2 = 10P_1 and P_2 values are required

We know that:P_1 = X_1/n_1P_1 = 20/80P_1 = 0.25P_2 = X_2/n_2P_2 = 10/100P_2 = 0.10

Hence, the values of P_1 and P_2 are 0.25 and 0.10 respectively.

Let n1 = 80, X1 = 20, n2 = 100, and X2 = 10P_1 and P_2 values are required

We know that:P_1 = X_1/n_1P_1 = 20/80P_1 = 0.25P_2 = X_2/n_2P_2 = 10/100P_2 = 0.10

Hence, the values of P_1 and P_2 are 0.25 and 0.10 respectively.

To know more about probability visit :-

https://brainly.com/question/31828911

#SPJ11

a) Find the general solution by solving the differential equation y'' + 2y' + y = 7 + 75 sin 2x using The Method of Undetermined Coefficients

b) Find the general solution by solving the differential equation y'' + y = sec(x) tan2(x) using The Method of Variation of Parameters

Answers

To find the general solutions of the given differential equations using different methods, we will use The Method of Undetermined Coefficients for the first equation and The Method of Variation of Parameters for the second equation.

The given differential equation is y'' + 2y' + y = 7 + 75 sin(2x). To solve this using The Method of Undetermined Coefficients, we assume the particular solution has the form yp = A + B sin(2x) + C cos(2x), where A, B, and C are constants. We then take the derivatives of yp and substitute them into the differential equation to solve for the coefficients. By adding the homogeneous solution yh = c1 e^(-x) + c2 x e^(-x), where c1 and c2 are constants, we obtain the general solution y = yp + yh.

The given differential equation is y'' + y = sec(x) tan²(x). To solve this using The Method of Variation of Parameters, we assume the particular solution has the form yp = u1(x) y1(x) + u2(x) y2(x), where y1(x) and y2(x) are linearly independent solutions of the homogeneous equation y'' + y = 0. We then find the Wronskian W = y1y2' - y1'y2, and the functions u1(x) and u2(x) are determined by integrating certain expressions involving the Wronskian and the given function in the differential equation.

Finally, by adding the homogeneous solution yh = c1 cos(x) + c2 sin(x), where c1 and c2 are constants, we obtain the general solution y = yp + yh. By applying these methods, we can find the general solutions of the given differential equations and obtain the complete set of solutions that satisfy the equations.

Learn more about differential equations here: brainly.com/question/25731911

#SPJ11

A given distribution function of some continuous random variable X:

F(x) = { 0, x<0
(a - 1)(1 - cos x), 0 < x ≤ π/2
1, x > π/2

a) Find parameter a;
b) Find the probability density function of the continuous random variable X;
c) Find the probability P(-π/2 ≤ x ≤ 1);
d) Find the median;
e) Find the expected value and the standard deviation of continuous random variable X.

Answers

a)  geta = 1 ; b) The probability density function f(x) = { 0, x ≤ 0 (a - 1) sin x, 0 < x ≤ π/2 0, x > π/2 ; c) Required probability is P(-π/2 ≤ x ≤ 1) = 1 ; d) M = π/2 - cos^(-1)(1/2a - 1) ; e) The standard deviation of the continuous random variable X is given by σ(X) = sqrt[(π² - 4) / 2].

Given distribution function of some continuous random variable X is given by

F(x) = { 0, x<0 (a - 1)(1 - cos x), 0 < x ≤ π/2 1, x > π/2a)

Find parameter

a;The given distribution function is given byF(x) = { 0, x<0 (a - 1)(1 - cos x), 0 < x ≤ π/2 1, x > π/2

To find the parameter a, use the property that a distribution function should be continuous and non decreasing.Here, the given distribution function is continuous and non decreasing at the point x = 0

Hence, the left hand limit and the right-hand limit of the distribution function at x = 0 should exist and they should be equal to 0.

Hence we have0 = F(0) = (a-1)(1 - cos 0) = (a-1)(1-1) = 0

So, we geta = 1

b) Find the probability density function of the continuous random variable X;The probability density function of a continuous random variable X is given by

f(x) = d/dxF(x) = d/dx {(a - 1)(1 - cos x)}, 0 < x ≤ π/2 = (a - 1) sin x, 0 < x ≤ π/2

The probability density function of the continuous random variable X is given by f(x) = { 0, x ≤ 0 (a - 1) sin x, 0 < x ≤ π/2 0, x > π/2

c) Find the probability P(-π/2 ≤ x ≤ 1);

Given distribution function F(x) = { 0, x<0 (a - 1)(1 - cos x), 0 < x ≤ π/2 1, x > π/2

Required probability is

P(-π/2 ≤ x ≤ 1) = F(1) - F(-π/2) = 1 - 0 = 1

d) Find the median;The median of a continuous random variable X is defined as that value of x for which the probability that X is less than x is equal to the probability that X is greater than x.

Mathematically,M = F^(-1)(1/2)

Thus, we have M = F^(-1)(1/2) = F^(-1)(F(M))

Solving for M, we get

M = π/2 - cos^(-1)(1/2a - 1)

The median of the continuous random variable X is given by

M = π/2 - cos^(-1)(1/2a - 1)

e) Find the expected value and the standard deviation of continuous random variable X.

The expected value of a continuous random variable X is given byE(X) = ∫xf(x)dx, -∞ < x < ∞

On substituting the value of f(x), we getE(X) = ∫(0 to π/2) x(a - 1) sin x dx = (a - 1) (π - 2)

On substituting the value of a = 1, we getE(X) = 0

The expected value of the continuous random variable X is given by E(X) = 0

The variance of a continuous random variable X is given byVar(X) = E(X²) - [E(X)]²

On substituting the value of f(x) and a, we getVar(X) = ∫(0 to π/2) x² sin x dx - 0= (π² - 4) / 2

On substituting the value of a = 1, we getVar(X) = (π² - 4) / 2

The standard deviation of the continuous random variable X is given by

σ(X) = sqrt[Var(X)]

On substituting the value of Var(X), we get

σ(X) = sqrt[(π² - 4) / 2]

Hence, the expected value of the continuous random variable X is 0, and the standard deviation of the continuous random variable X is given by σ(X) = sqrt[(π² - 4) / 2].

Know more about the probability density function

https://brainly.com/question/15714810

#SPJ11

8. Which of the correlation coefficients shown below indicates the strongest linear correlation? a) - 0.903 b) 0.720 c) -0.410 d) 0.203 9. A manager of the credit department for an oil company would l

Answers

Based on this, the correlation coefficient that indicates the strongest linear correlation is -0.903 which is option A.

Correlation coefficient is a statistical measure that indicates the extent to which two or more variables change together. The correlation coefficient ranges from -1 to +1.

If the correlation coefficient is +1, there is a perfect positive relationship between the variables. When the correlation coefficient is -1, there is a perfect negative correlation between the variables.

A strong positive linear correlation is indicated by a correlation coefficient that is close to +1. While a strong negative linear correlation is indicated by a correlation coefficient that is close to -1. A correlation coefficient of 0 indicates no correlation between the two variables.

This indicates a strong negative linear correlation.9.

A manager of the credit department for an oil company would like to determine whether there is a linear relationship between the amount of outstanding receivables (in thousands of dollars) and the size of the firm (in millions of dollars). The best tool for this analysis is linear regression.

Linear regression is a statistical method that examines the relationship between two continuous variables. It can be used to determine if there is a relationship between the two variables and to what extent they are related. Linear regression calculates the line of best fit between the two variables.

This line can then be used to predict the value of one variable based on the value of the other variable.

To know more about correlation coefficients visit:

https://brainly.com/question/29978658

#SPJ11

Solve the matrix equation for X: X [ 1 -1 2] = [-27 -3 0]
[5 0 1] [ 9 -4 9]
X =

Answers

The matrix equation for X: X [ 1 -1 2] = [-27 -3 0], X = [-27 -3 0; 9 -4 9] * [1 -1 2; 5 0 1]⁻¹

To solve the matrix equation X [1 -1 2] = [-27 -3 0; 9 -4 9], we first need to find the inverse of the matrix [1 -1 2; 5 0 1]. The inverse of a 2x3 matrix is a 3x2 matrix. In this case, the inverse is [-2/7 2/7; 5/7 -1/7; 8/7 -1/7].

Next, we multiply the given matrix [-27 -3 0; 9 -4 9] by the inverse matrix [1 -1 2; 5 0 1]⁻¹. Performing this multiplication gives us the final solution for X. The resulting matrix equation is X = [-1 -2 2; 1 -1 0].

Learn more about matrix equation here: brainly.com/question/27572352

#SPJ11

systematic error is defined as group of answer choices error that is random. error that has equal probability of being too high and too low. error that averages out with repeated trials. error that tends to be too high or too low.

Answers

Error that tends to be too high or too low is defined as a systematic error. Avoiding observational errors - it is vital to be meticulous and record the readings accurately.

Systematic errors are those errors that are consistent and can be reliably replicated under the same conditions. These errors are not random and are mostly caused by the faulty apparatus used to perform the experiment. These errors tend to produce measurements that are consistently too high or too low from the true value.

The outcomes of random errors can be either too high or too low, and they usually balance out over multiple trials. In contrast, systematic errors are consistent and can be accounted for by performing a correction factor on the measurement.

These errors can lead to skewed results and can cause an experiment to be inaccurate and unreliable.

To know more about systematic error visit :-

https://brainly.com/question/30779771

#SPJ11

Find a + b, a - b, 4a + 5b, 4a - 5b, and ||a||.
a = -(3, -6), b = 3(0, -6)
a + b =_____
a - b =______
4a + 5b =______
4a - 5b =______
||a|| = _______

Answers

Given vectors a = -(3, -6) and b = 3(0, -6), we can compute the vector operations. The results are as follows: a + b = (0, -12), a - b = (-6, 0), 4a + 5b = (-12, -90), 4a - 5b = (6, 78), and ||a|| = 6.

To compute vector addition, we add the corresponding components of the vectors. a + b = (-3 + 0, -6 + (-18)) = (0, -24).

For vector subtraction, we subtract the corresponding components. a - b = (-3 - 0, -6 - (-18)) = (-3, 12).

To find the scalar multiplication, we multiply each component of the vector by the scalar. 4a + 5b = 4(-3, -6) + 5(0, -18) = (-12, -24) + (0, -90) = (-12 + 0, -24 + (-90)) = (-12, -114).

Similarly, 4a - 5b = 4(-3, -6) - 5(0, -18) = (-12, -24) - (0, -90) = (-12 - 0, -24 - (-90)) = (-12, 66).

The magnitude of a vector, denoted as ||a||, is computed using the formula ||a|| = √(a₁² + a₂²). For vector a = (-3, -6), ||a|| = √((-3)² + (-6)²) = √(9 + 36) = √45 = 6.

In summary, a + b = (0, -12), a - b = (-6, 0), 4a + 5b = (-12, -90), 4a - 5b = (6, 78), and ||a|| = 6.

To learn more about vectors click here :

brainly.com/question/31737683

#SPJ11

Let X₁, X₂.... 2022/05/2represent a random sample from a shifted exponential with pdf f(x; λ,0) = Ae-(-0); x ≥ 0, > where, from previous experience it is known that 0 = 0.64. a. Construct a maximum-likelihood estimator of A. b. If 10 independent samples are made, resulting in the values: 3.11, 0.64, 2.55, 2.20, 5.44, 3.42, 10.39, 8.93, 17.82, and 1.30 calculate the estimates of A.

Answers

(a) Construct a maximum-likelihood estimator of A:

To construct the maximum-likelihood estimator of A, we need to maximize the likelihood function based on the given sample. The likelihood function L(A) is defined as the product of the probability density function (PDF) evaluated at each observation.

Given that the PDF is f(x; λ, 0) = Ae^(-λx), where x ≥ 0, and we have a sample of independent observations X₁, X₂, ..., Xₙ, the likelihood function can be written as:

L(A) = A^n * e^(-λΣxi)

To maximize the likelihood function, we can take the natural logarithm of both sides and find the derivative with respect to A, and set it equal to zero.

ln(L(A)) = nln(A) - λΣxi

Taking the derivative with respect to A and setting it equal to zero, we get:

d/dA ln(L(A)) = n/A - 0

n/A = 0

n = 0

Therefore, the maximum-likelihood estimator of A is A = n.

(b) Given the sample values: 3.11, 0.64, 2.55, 2.20, 5.44, 3.42, 10.39, 8.93, 17.82, and 1.30, we have n = 10.

Hence, the estimate of A is A = n = 10.

To know more about Function visit-

brainly.com/question/31062578

#SPJ11

7. Given the lines L₁: (x, y, z) = (1, 3,0) + t(4, 3, 1), L₂: (x, y, z) = (1, 2, 3 ) + t(8, 6, 2 ),
the plane P: 2x − y + 3z = 15 and the point A(1, 0, 7 ).
a) Show that the lines L₁ and L₂ lie in the same plane and find the general equation of this plane.
b) Find the distance between the line L₁ and the Y-axis.
c) Find the point Bon the plane P which is closest to the point A.

Answers

Answer:

a) To show that the lines L₁ and L₂ lie in the same plane, we can demonstrate that both lines satisfy the equation of the given plane P: 2x - y + 3z = 15.

For Line L₁:

The parametric equations of L₁ are:

x = 1 + 4t

y = 3 + 3t

z = t

Substituting these values into the equation of the plane:

2(1 + 4t) - (3 + 3t) + 3t = 15

2 + 8t - 3 - 3t + 3t = 15

7t - 1 = 15

7t = 16

t = 16/7

Therefore, Line L₁ satisfies the equation of plane P.

For Line L₂:

The parametric equations of L₂ are:

x = 1 + 8t

y = 2 + 6t

z = 3 + 2t

Substituting these values into the equation of the plane:

2(1 + 8t) - (2 + 6t) + 3(3 + 2t) = 15

2 + 16t - 2 - 6t + 9 + 6t = 15

16t + 6t + 6t = 15 - 2 - 9

28t = 4

t = 4/28

t = 1/7

Therefore, Line L₂ satisfies the equation of plane P.

Since both Line L₁ and Line L₂ satisfy the equation of plane P, we can conclude that they lie in the same plane.

The general equation of the plane P is 2x - y + 3z = 15.

b) To find the distance between Line L₁ and the Y-axis, we can find the perpendicular distance from any point on Line L₁ to the Y-axis.

Consider the point P₁(1, 3, 0) on Line L₁. The Y-coordinate of this point is 3.

The distance between the Y-axis and point P₁ is the absolute value of the Y-coordinate, which is 3.

Therefore, the distance between Line L₁ and the Y-axis is 3 units.

c) To find the point B on plane P that is closest to the point A(1, 0, 7), we can find the perpendicular distance from point A to plane P.

The normal vector of plane P is (2, -1, 3) (coefficient of x, y, z in the plane's equation).

The vector from point A to any point (x, y, z) on the plane can be represented as (x - 1, y - 0, z - 7).

The dot product of the normal vector and the vector from point A to the plane is zero for the point on the plane closest to point A.

(2, -1, 3) · (x - 1, y - 0, z - 7) = 0

2(x - 1) - (y - 0) + 3(z - 7) = 0

2x - 2 - y + 3z - 21 = 0

2x - y + 3z = 23

Therefore, the point B on plane P that is closest to point A(1, 0, 7) lies on the plane with the equation 2x - y + 3z = 23.

When Emma saves each month for a goal, what is the value of the goal called?

A.
annuity value
B.
future value
C.
payment value
D.
present value

Answers

When Emma saves each month for a goal, the value of the goal called is referred to as (B) future value.

An annuity is a stream of equal payments received or paid at equal intervals of time. Annuity value represents the present value of the annuity amount that will be received at the end of the specified time period. Future value (FV) is the value of an investment after a specified period of time. It is the value of the initial deposit plus the interest earned on that deposit over time. The future value of a single deposit will increase over time due to the effect of compounding interest.

When Emma saves each month for a goal, the amount she saves accumulates over time and earns interest. The future value is calculated based on the initial deposit amount, the number of months it will earn interest, and the interest rate. It is important to determine the future value of the goal in order to make effective financial decisions that will enable Emma to achieve her goal.

To know more about annuity value visit:

https://brainly.com/question/21801625

#SPJ11

Find the greatest common factor of 11n and 14c.

Answers

The greatest common factor of 11n and 14c is 1. This means that there is no number greater than 1 that can divide both 11n and 14c without leaving a remainder.

To find the greatest common factor (GCF) of 11n and 14c, we need to determine the largest number that divides both 11n and 14c without leaving a remainder.

Let's break down the two terms: 11n and 14c. The prime factorization of 11 is 11, which means it is a prime number and cannot be further factored. Similarly, the prime factorization of 14 is 2 × 7.

Since the GCF must have factors common to both terms, the common factors between 11n and 14c are the factors they share. In this case, the only factor they have in common is 1.

Learn more about greatest common factor :

https://brainly.com/question/22290900

#SPJ11

The patient recovery time from a particular surgical procedure is normally distributed with a mean of 5.7 days and a standard deviation of 2.5 days. What is the 90th percentile for recovery times? (Round your answer to two decimal places

Answers

The patient recovery time from a particular surgical procedure is normally distributed with a mean of 5.7 days

and a standard deviation of 2.5 days. What is the 90th percentile for recovery times? (Round your answer to two decimal places)For a normal distribution, we have the z score that can be computed as follows:z = (x - μ) / σwherez = the standard scorex = the raw scoreμ = the meanσ = the standard deviation

The formula for finding the percentile from the standard score is:Percentile = (1 - z) × 100The given information is that the mean is 5.7 and the standard deviation is 2.5, hence for the 90th percentile, the value of the standard score is:z90 = 1.28To determine the value of x corresponding to this z score, we substitute into the formula:z = (x - μ) / σ1.28 = (x - 5.7) / 2.5Multiplying through by 2.5 gives:x - 5.7 = 3.2x = 8.9Therefore, the 90th percentile for recovery times is 8.9 days (rounded to two decimal places).

To know more about equation visit:

https://brainly.com/question/649785

#SPJ11

Question 15 1 pts A pair of standard 6-sided number cubes are rolled. Rank the following outcomes from most likely to least likely. • X = rolling a 2 . Y = rolling a 7 . Z = rolling a 10 OZ.XY OZ.Y.X OY,Z,X O Y.X, Z

Answers

Ranking from most likely to least likely: OY.X,Z, OY,Z,X, OZ.Y.X, OZ.XY. Rolling a 7 is more likely than rolling a 2 or 10, while rolling a 10 is less likely overall.

 

In this case, rolling a pair of standard 6-sided number cubes means that each cube has six possible outcomes (numbers 1 to 6). Let's analyze the outcomes:

1. OZ.XY: This outcome represents rolling a 10 first and then rolling a 2. Since the maximum possible sum of two dice is 12 (6+6), rolling a 10 is less likely than rolling a 2. Therefore, OZ.XY is the least likely outcome.

2. OZ.Y.X: This outcome represents rolling a 10 first, followed by rolling a 7. Similarly to the previous case, rolling a 10 is less likely than rolling a 7. Therefore, OZ.Y.X is the second least likely outcome.

3. OY,Z,X: This outcome represents rolling a 7 first, then rolling a 10, and finally rolling a 2. Rolling a 7 is more likely than rolling a 10 or a 2 since there are multiple ways to obtain a sum of 7 (1+6, 2+5, 3+4, 4+3, 5+2, 6+1). Therefore, OY,Z,X is the second most likely outcome.

4. OY.X,Z: This outcome represents rolling a 7 first, then rolling a 2, and finally rolling a 10. Similar to the previous case, rolling a 7 is more likely than rolling a 2 or a 10. Therefore, OY.X,Z is the most likely outcome.

So, the ranking from most likely to least likely is as follows:

1. OY.X,Z

2. OY,Z,X

3. OZ.Y.X

4. OZ.XY

To learn more about ranking click here

brainly.com/question/31689723

#SPJ11

Treating circulatory disease: Angioplasty is a medical procedure in which an obstructed blood vessel is widened. In some cases, a wire mesh tube, called a stent, is placed in the vessel to help it remain open. A study was conducted to compare the effectiveness of a bare metal stent with one that has been coated with a drug designed to prevent reblocking of the vessel. A total of 5312 patients received bare metal stents, and of these, 832 needed treatment for reblocking within a year. A total of 1112 received drug-coated stents, and 121 of them required treatment within a year. Can you conclude that the proportion of patients who needed retreatment differs between those who received bare metal stents and those who received drug-coated stents? Lep 1 denote the proportion of patients with bare metal stents who needed retreatment. Use the = 0.10 level and the critical value method with the table.

Part 1 out of 5
State the appropriate null and alternate hypotheses.
Part 2: How many degrees of freedom are there, using the simple method?
Part 3: Find the critical values. Round three decimal places.
Part 4: Compute the test statistic. Round three decimal places.

Answers

1. Null Hypotheses :H0: p1 = p2 ; Alternate Hypotheses :Ha: p1 ≠ p2 ; 2. df = 6422 ; 3.The critical values are ±1.645. ; 4. the test statistic is 2.747.

Part 1: State the appropriate null and alternate hypotheses.The appropriate null and alternate hypotheses for the given information are as follows:

Null Hypotheses:H0: p1 = p2

Alternate Hypotheses:Ha: p1 ≠ p2

Where p1 = proportion of patients who received bare metal stents and needed retreatment, and p2 = proportion of patients who received drug-coated stents and needed retreatment.

Part 2: How many degrees of freedom are there, using the simple method? The degrees of freedom (df) can be found using the simple method, which is as follows:df = n1 + n2 - 2

Where n1 and n2 are the sample sizes of the two groups .n1 = 5312

n2 = 1112

df = 5312 + 1112 - 2 = 6422

Part 3: Find the critical values. Round three decimal places.

The level of significance is α = 0.10, which means that α/2 = 0.05 will be used for a two-tailed test.The critical values can be found using a t-distribution table with df = 6422 and α/2 = 0.05. The critical values are ±1.645.

Part 4: Compute the test statistic. Round three decimal places.The test statistic can be calculated using the formula:z = (p1 - p2) / √[p(1 - p) x (1/n1 + 1/n2)]

Where p = (x1 + x2) / (n1 + n2), x1 and x2 are the number of patients who needed retreatment in each group.

x1 = 832, n1 = 5312, x2 = 121, n2 = 1112p = (832 + 121) / (5312 + 1112) = 0.138z = (0.147 - 0.109) / √[0.138(1 - 0.138) x (1/5312 + 1/1112)]≈ 2.747

Therefore, the test statistic is 2.747.

Know more about the Null Hypotheses

https://brainly.com/question/4436370

#SPJ11

Maximize z = x + 3y, subject to the constraints: x + y ≤ 4, x ≥ 0, y ≥ 0, find the maximum value of z ? a. 0 b. 4 c. 12 d. 16

Answers

The correct option is c. The maximum value of z is 12. To find the maximum value of the objective function z = x + 3y, subject to the given constraints x + y ≤ 4, x ≥ 0, and y ≥ 0, we need to optimize the objective function within the feasible region defined by the constraints.

The feasible region is defined by the inequalities x + y ≤ 4, x ≥ 0, and y ≥ 0. Graphically, it represents the area below the line x + y = 4 and bounded by the x and y axes.

To find the maximum value of z = x + 3y within this feasible region, we can examine the corner points of the region. These corner points are (0, 0), (0, 4), and (4, 0).

Substituting the coordinates of each corner point into the objective function, we find:

- For (0, 0): z = 0 + 3(0) = 0

- For (0, 4): z = 0 + 3(4) = 12

- For (4, 0): z = 4 + 3(0) = 4

Among these values, the maximum value of z is 12, which corresponds to the point (0, 4) within the feasible region.

Hence, the correct option is c. The maximum value of z is 12.

To learn more about inequalities , click here;

brainly.com/question/20383699

#SPJ11

Use Simpson's rule with n = 10 to approximate
∫5 1 cos(x)/x dx
Keep at least 2 decimal places accuracy in your final answer

Answers

We want to calculate the value of the definite integral $\int_{1}^{5} \frac{\cos(x)}{x} dx$ using Simpson's rule with n=10.

First, we have to calculate the interval width of each segment, which is given by $\Delta x = \frac{5-1}

{10}=0.4$Next, we calculate the values of the function at the endpoints of the intervals.Using the left endpoints for the first four segments, we get:$f(1) = \frac{\cos(1)}{1}=0.5403$ $f

(1.4) = \frac{\cos(1.4)}{1.4}=0.4077$ $

f(1.8) = \frac{\cos(1.8)

}{1.8}=0.3126$

$f(2.2) = \frac{\cos(2.2)}

{2.2}=0.2394$Using the midpoints for the next five segments, we get:$f(2.6) = \frac{\cos(2.6)}

{2.6}=0.1885$ $f(3.0) = \frac{\cos(3.0)}

{3.0}=0.1310$

$f(3.4) = \frac{\cos(3.4)}

{3.4}=0.0899$

$f(3.8) = \frac{\cos(3.8)}

{3.8}=0.0627$

$f(4.2) = \frac{\cos(4.2)}

{4.2}=0.0449$Using the right endpoint for the last segment, we get:$f(4.6) = \frac{\cos(4.6)}

{4.6}=0.0323$Next, we can apply Simpson's rule:$$\begin{aligned}\int_{1}^{5} \frac{\cos(x)}{x} dx &\approx \frac{\Delta x}{3}\left[f(1)+4f(1.4)+2f(1.8)+4f(2.2)+2f(2.6)+4f(3.0) \right.\\&\quad \left. +2f(3.4)+4f(3.8)+2f(4.2)+f(4.6)\right]\\&= \frac{0.4}{3}\left[0.5403+4(0.4077)+2(0.3126)+4(0.2394)+2(0.1885)\right.\\&\quad \left. +4(0.1310)+2(0.0899)+4(0.0627)+2(0.0449)+0.0323\right]\\&= 0.3811\end{aligned}$$Rounding to two decimal places, the final answer is 0.38. Therefore, $\int_{1}^{5} \frac{\cos(x)}{x} dx \approx 0.38$.

To know more about area visit:

https://brainly.com/question/13194650

#SPJ11

A
random sample of 117 lighting flashes in a certain region resultef
in a sample average radar exho duration of 0.80 sec and a sample
deviation of 0.49 sec. Calculate a 99%( two sided) confidence
inte
DETAILS DEVORESTATS 7.5.01.XP kang mingle average ratar w amers by bat da ped the in f the plain led the pl population means is interd Ma m may read the late in the Appends of Talent qu o [ "plakjes v

Answers

Random sample of 117 lighting flashes in a certain region resulted in a sample average radar echo duration of 0.80 sec and a sample deviation of 0.49 sec.

option B is correct.

We have to Calculate a 99%( two-sided) confidence interval.**Solution:**Let $\bar{x}$ be the sample mean radar echo duration.Then the 99% confidence interval for population mean radar echo duration is given by:$\bar{x} - z_{\frac{\alpha}{2}} \frac{\sigma}{\sqrt{n}} < \mu < \bar{x} + z_{\frac{\alpha}{2}} \frac{\sigma}{\sqrt{n}}$Where,

$n = 117$,

sample size$\bar{x} = 0.80$,

sample mean$\sigma = 0.49$,

sample deviation$\alpha = 0.01$,

confidence level$z_{\frac{\alpha}{2}} = z_{0.005}$,

from normal distribution table$z_{0.005} = 2.58$Substitute the given values in the above expression,

we get:$$\begin{aligned}\bar{x} - z_{\frac{\alpha}{2}} \frac{\sigma}{\sqrt{n}} &< \mu < \bar{x} + z_{\frac{\alpha}{2}} \frac{\sigma}{\sqrt{n}}\\\frac{4}{5} - (2.58) \frac{0.49}{\sqrt{117}} &< \mu < \frac{4}{5} + (2.58) \frac{0.49}{\sqrt{117}}\\0.744 &< \mu < 0.856\end{aligned}$$Hence, the required 99% confidence interval for population mean radar echo duration is $(0.744, 0.856)$.

To know more about sample average visit:

https://brainly.com/question/31419935

#SPJ11

An insurer has 10 separate policies with coverage for one year. The face value of each of those policies is $1,000.
The probability that there will be a claim in the year under consideration is 0.1. Find the probability that the insurer will pay out more than the expected total for the year under consideration.

Answers

Let X be the random variable for the total payout. Then we can say that $X$ is the sum of the payouts of the 10 policies. As there are 10 policies and the face value of each policy is $1000, the total expected payout would be $10,000.The probability of there being a claim is given as 0.1. Hence the probability of there not being a claim would be 0.9. This is important to know as it helps us calculate the probability of paying out more than the expected total for the year under consideration.

Let's find the standard deviation for the variable X.σX = √(npq)σX = √(10 × 1000 × 0.1 × 0.9)σX = 94.87

Therefore, the expected value and standard deviation of the total payout are:

Expected value = μX = np = 1000 × 10 × 0.1 = $1000

Standard deviation = σX = 94.87Using the Chebyshev’s theorem, we can say:P(X > E(X) + kσX) ≤ 1/k²

The insurer is an individual who gives protection to people for financial losses or damages in the form of a policy.

Here we calculated the probability of an insurer paying more than the expected total for the year under consideration.

The probability of a claim is given as 0.1.

Hence the probability of there not being a claim would be 0.9. Using the Chebyshev’s theorem, we found out that the probability of paying out more than the expected total for the year under consideration is ≤ 0.25.

learn more about probability click here:

https://brainly.com/question/13604758

#SPJ11

Solve the absolute value inequality. Write the solution in interval notation. 3|x-9|+9<15 Select one:
a. (-[infinity], 7) U (11,[infinity]) b. (-[infinity], 1) U (17,[infinity]) c. (7. 11) d. (1.17)

Answers

The solution to the absolute value inequality 3|x-9|+9<15 is option d. (1,17).

To solve the absolute value inequality 3|x-9|+9<15, we need to isolate the absolute value expression and consider both the positive and negative cases.

First, subtract 9 from both sides of the inequality:

3|x-9| < 6

Next, divide both sides by 3:

|x-9| < 2

Now, we consider the positive and negative cases:

Positive case:

For the positive case, we have:

x-9 < 2

Solving for x, we get:

x < 11

Negative case:

For the negative case, we have:

-(x-9) < 2

Expanding and solving for x, we get:

x > 7

Combining both cases, we have the solution:

7 < x < 11

Expressing the solution in interval notation, we get option d. (1,17), which represents the open interval between 1 and 17, excluding the endpoints.

Learn more about absolute value inequality here: brainly.com/question/30201926

#SPJ11

Use the discriminant to determine the type and number of solutions. -2x² + 5x + 5 = 0 Select one: a. One rational solution O b. Two imaginary solutions Oc. Two rational solutions d. Two irrational solutions

Answers

The given quadratic equation is 3x^2 - 4x - 160 = 0.

To find the solutions of the quadratic equation, we can use the quadratic formula:

x = (-b ± sqrt(b^2 - 4ac)) / (2a)

In this equation, a = 3, b = -4, and c = -160. Substituting these values into the quadratic formula, we get:

x = (-(-4) ± sqrt((-4)^2 - 4 * 3 * (-160))) / (2 * 3)

Simplifying further:

x = (4 ± sqrt(16 + 1920)) / 6

x = (4 ± sqrt(1936)) / 6

x = (4 ± 44) / 6

We have two possible solutions:

x = (4 + 44) / 6 = 48 / 6 = 8

x = (4 - 44) / 6 = -40 / 6 = -20/3

Therefore, the solutions to the quadratic equation 3x^2 - 4x - 160 = 0 are x = 8 and x = -20/3.

Now, let's analyze the quadratic equation and its solutions. Since we are dealing with a real quadratic equation, it is possible to have real solutions. In this case, we have two real solutions: one is a rational number (8) and the other is an irrational number (-20/3).

The rational solution x = 8 indicates that there is a point where the quadratic equation intersects the x-axis. It represents the x-coordinate of the vertex of the parabolic graph.

The irrational solution x = -20/3 indicates another point of intersection with the x-axis. It represents another possible value for x that satisfies the equation.

Learn more about rational here : brainly.com/question/15837135

#SPJ11

Write the logarithmic expression as a single logarithm with a coefficient of 1. 4(log3 7 + log3 y) - log3 z

Answers

The required  logarithmic expression is log3 [(7^4 × y^4)/z] if coefficient   1. 4(log3 7 + log3 y) - log3 z.

Let's first express the given logarithmic expression as a single logarithm with a coefficient of 1.

Step 1: Simplify the given expression.4(log3 7 + log3 y) - log3 z= log3 (7^4 × y^4) - log3 z

Step 2: Use the following logarithmic identity.

If logb M - logb N, then logb (M/N).4(log3 7 + log3 y) - log3 z= log3 [(7^4 × y^4)/z]

The expression 4(log3 7 + log3 y) - log3 z can be written as a single logarithm with a coefficient of 1 as log3 [(7^4 × y^4)/z].

To know more about logarithmic expression Visit:

https://brainly.com/question/29194783

#SPJ11

raster data model is widely used to represent field features, but cannot represent point, line, and polygon features.

Answers

The raster data model is commonly used to represent field features, but it is not suitable for representing point, line, and polygon features.

The raster data model is a grid-based representation where each cell or pixel contains a value representing a specific attribute or characteristic. It is well-suited for representing continuous spatial phenomena such as elevation, temperature, or vegetation density. Raster data is organized into a regular grid structure, with each cell having a consistent size and shape.

However, the raster data model has limitations when it comes to representing discrete features like points, lines, and polygons. Since raster data is based on a grid, it cannot precisely represent the exact shape and location of these features. Instead, they are approximated by the cells that cover their extent.

Learn more about polygons here:

https://brainly.com/question/23846997

#SPJ11

Find the 17th term of the geometric sequence if a₅, -64 and a₈ = 91.

Answers

The 17th term of the geometric sequence is -4,096.

To find the 17th term of the geometric sequence, we need to determine the common ratio (r) first. We can do this by dividing the 8th term (a₈ = 91) by the 5th term (a₅).

r = a₈ / a₅

r = 91 / (-64)

r = -1.421875

Now that we have the common ratio, we can use it to find the 17th term (a₁₇) by multiplying the 8th term by the common ratio raised to the power of the number of terms between the 8th and 17th term, which is 9.

a₁₇ = a₈ * (r)⁹

a₁₇ = 91 * (-1.421875)⁹

a₁₇ ≈ -4,096

Therefore, the 17th term of the geometric sequence is -4,096.

Learn more about geometric sequence here: brainly.com/question/27852674

#SPJ11

a. Construct a scatterplot. b. Find the value of the linear correlation coefficient r, then determine whether there is suffi- cient evidence to support the claim of a linear correlation between the tw

Answers

Plot the above data on a graph by taking x-axis as independent variable and y-axis as dependent variable: The value of the linear correlation coefficient (r) between the two variables X and Y is 0.611.


To support the claim of a linear correlation between the two variables:
We will use the following formula to calculate the linear correlation coefficient (r) between the two variables:
r = n∑XY − (∑X)(∑Y) / {√[n∑X² − (∑X)²][n∑Y² − (∑Y)²]}

So, the value of the linear correlation coefficient (r) between the two variables X and Y is 0.611.So, there is sufficient evidence to support the claim of a linear correlation between the two variables.

To know more about variable visit :-

https://brainly.com/question/15078630

#SPJ11

A furniture manufacturer took 68 hours to make the first premium elegance chair. The factory is known to have a 75% learning curve. How long will it take to make chair number 13 only. Select one: O a. 23.46 hours O b. 20.98 hours O c. 70.00 hours O d. Oe. Time left 1:13:33 none of the listed answers 452.28 hou

Answers

According to the 75% learning curve, it is estimated that it will take approximately 23.46 hours to manufacture chair number 13.

The learning curve is a concept that suggests the time required to complete a task decreases as the cumulative volume of production increases. In this case, the learning curve is stated to be 75%, which means that for each doubling of the cumulative volume of production, the time required decreases by 25%.

To determine the time it will take to manufacture chair number 13, we need to calculate the learning curve rate. The formula to calculate the learning curve rate is as follows:

Learning Curve Rate = log(learning curve percentage) / log(2)

In this case, the learning curve rate is calculated as:

Learning Curve Rate = log(75%) / log(2) ≈ -0.415

Next, we can use the learning curve formula to find the time required for chair number 13. The formula is:

Time required for a specific unit = Time required for the first unit × (Cumulative volume of production for the specific unit)^learning curve rate

Given that the first premium elegance chair took 68 hours to manufacture, and we want to find the time for chair number 13, the calculation is:

Time required for chair number 13 = 68 × ([tex]13^{(-0.415)[/tex]) ≈ 23.46 hours

Therefore, it is estimated that it will take approximately 23.46 hours to manufacture chair number 13, which corresponds to option (a) in the provided choices.

Learn more about learning curve here:

https://brainly.com/question/31721052

#SPJ11

Other Questions
You are standing at the top of a 150 m tall tower and throw a 2 kg rock straight up at 10 m/s. A friend of yours throws a rock with the same mass straight down at the same. speed. Which rock, if either, has a greater speed when it reaches the ground? It is acceptable to answer this question without a direct calculation, but be sure to clearly and fully explain your reasoning, addressing this learning target (1D kinematics), if you do so. Based on what you know so far about leadership, who is a person that you know, that you would consider a leader. Why do you think so try to apply some definition and description from the text book to describe their leadership style. self confidence, Humility, core self evaluation, sense of humor, trustworthiness, Authenticity, Extraversion, Assertiveness, Euthusiasm, optimism and warmth. problem solving skills. A stock has a beta of 2.57 and an expected return of 28.0%. The T-bill rate is 7.6%. Calculate the portfolio beta if the expected return of a portfolio of the two assets is 19.8%. Answer: Site staff may begin recruitment upon completion of all action items required for endorsement, whenadequate staff is available and fully trained in their study roles, and after.O A. The research site investigator sends a letter to the sponsor advising of their site-appointed start date.O B. The research site has been inspected and received approval from an FDA Inspector.OC. The Lead Investigator and sponsor have provided their approval.OD. None of the above. how long does it take to see symptoms of thiamin deficiency after the diet has become deficient? Participate in a discussion regarding the production possibilities frontier, which is a model of scarcity, choice, and opportunity cost, Apply this model to your life by describing your forgone leisure or forgone income as your tradeoff for a higher GPA score. A lower grade is the opportunity cost of increased leisure or increased income. Also identify your position point on the PPF and indicate how many study hours you're planning to set aside to complete your coursework in this class. Write your answer to at least 3 decimal places if appropriate, unless it is specified otherwise. 1. Let u be the mean level of Selenium in the blood for elderly people (in units mg/dL). It is of interest to know whether these mean levels have changed over time from the historical value of Selenium. It is claimed that due to a change in diet, the mean level of Selenium in the blood for elderly people (in units mg/dL), H, is no longer equal to 19.6 but has increased. Let X denote the level of Selenium in the blood of a random selected elderly person. A random sample of Selenium from n = 21 individuals is taken from the population of elderly people. The following summary statistics are obtained from the sample: n sample mean sample sd 21 22.1889 4.225254 We can assume each observation is independent and identically distributed N (u,0%). Carry out a one-sample t-test : and complete the exercises below. In this question, t(4) = t4, at distribution with 4 degrees of freedom. (a) Select the null distribution of the test statistic. That is, the distribution of the test statistic assuming He is true. a)t(20) b)N (0,1) c)t(10) d)t(21) e) t(22) (b) Compute the observed value of the test statistic for this hypothesis test. Write your answer to at least 3 decimal places. (c) P-value for this hypothesis test lies in which of the following interval? (0.1,1) 0(0.05, 0.1) (0.025, 0.05) O(0.01, 0.025) 0(0, 0.01) There is a line passing through P = (2, 1, 5) and Q = (7, -2,4). (a) Write the vector equation of the line described. (b) Write the parametric equation of the line described. (c) Write the symmetric equation of the line described. Consider a firm that uses capital, K, to invest in a project that generates revenue and the MR from the 1st, 2nd, 3rd, 4th & 5th unit of K is $1.75, 1.48, 1.26, 1.18 and 1.13. respectively. (This is just MR table, as in the notes). If the interest rate is 21% , then. the optimal K for the firm to borrow is - 2 - 3 - 4 - 5 Solve the following 0-1 integer programming model problem by implicit enumeration. Maximize 4x + 5x2 + x3 + 3x4 + 2x5 + 4x6 + 3x7 + 2x8 + 3x9 Subject to 3x2 + x4 + X5 23 x + x 1 X2 + X4 X5 X6 -1 x + 2x + 3x7 + x8 + 2x9 4 -x3 + 2x5 + X6 + 2x72x8 + x9 5 X1, X2, X3, X4, X5, X6, X7, X8, X9 {0,1} The total cost (in dollars) to desalinate tons of salt water every week is given byC(x) 700+100x-100 In(x), x 1Find the minimum average cost Minimum Average Cost = dollars per ton GROUP ASSIGNMENT How to measure HR's contribution? -Choose an industry -Write about metrics which should be used in that industry and your explanation What did Le Texier find after reviewing archived records of Zimbardo's Stanford Prison Experiment?A) Two guards and one prisoner later said their behavior was an act.B) Three of Zimbardo's associates repeatedly challenged his methodology.C) The guards had been given virtually no instructions on how to treat the prisoners.D) That the completely immersive nature of the experiment biased the results. a projectile has a height given by the function h(t)=-4.9(t-4)^2 153 where time,t, is in seconds and the height, h, is in meters. What is the maximum height of the function and at what time does it reach thatheight? So if we have a tobin's q greater than 1 (vs. less than one), should the firm use retained earnings to do share buyback or just pay out dividends? Show the reasoning when to give out dividends or when to do share buyback, depending on tobin's q. 32. the anterior part of the pituitary gland secretes which hormones? At some vacation destinations, "all-inclusive" resorts allow you to pay a flat rate and then eat and drink as much as you want. There has been concern about whether these deals might lead to excessive consumption of alcohol by young adults on spring break trips. You decide to spend your spring break collecting data on this issue. Of course, you need to take all your friends on this funded research trip, because you need a lot of research assistants! You collect data on the number of drinks consumed in a day by people staying at all-inclusive resorts and by those staying at noninclusive resorts. Your data are represented below. All-inclusive resort guests: 10 8 13 9 11 Noninclusive resort guests: 3 15 7 8 10 Do guests at all-inclusive resorts consume a significantly different amount of alcohol compared to guests at noninclusive resorts? State the null and research (alternative) hypotheses in words and using symbols. Conduct the appropriate hypothesis test with a = .05 and state your conclusion in terms of this problem. Remember to use the 4 steps of hypothesis testing and include the SPSS output as evidence for calculations. as an approach to structuring families and communities, patriarchy is virtually universal.True or false Company has no debt. It has $40 million in common stock outstanding and $10 million in preferred stock. Its corporate tax rate is 30%. Its common stockholders expect a return of 17%. Its preferred stockholders expect a return of 12%. What is Company's weighted average cost of capital?A. 16%B. 11.92%C. 15.28%D. 17%E. None of the above how many official ports of entry are there in the united states?